0 Daumen
2,8k Aufrufe

Bild Mathematik Wie fange ich bei dieser Aufgabe an ? Wie strukturiere ich sie wie löse ich sie ?

Liebe Grüße

Bild im Anhang

von

4 Antworten

0 Daumen

Fläche des Parallelogramms

A = 8 * 5 - x * (8 - x) - x * (5 - x) = 2·x^2 - 13·x + 40

Extremstelle A'(x) = 0

A' = 4·x - 13 = 0 --> x = 3.25

von 477 k 🚀
0 Daumen

Mach dir Gedanken wie du das Parallelogramm beschreiben kannst, bzw. dessen Größe:

Vllt:

Fläche Rechteck-Fläche der vier Randreecke


(5*8)-(2*(1/2*(8-x))*x)-(2*(1/2*(5-x))*x)

-> 40 - (x(8-x+5-x))

-> 2x2-13x+40

Für den kleinsten Flächeninhalt, suchst du Minima der Funktion.

, d.h ableiten, notwendige Bedingung -> hinreichende Bedingung.

oder

einfach den Scheitelpunkt bestimmen



Zur Kontrolle:

der Flächeninhalt ist für x= 13/4= 3,25 am kleinsten

von 8,7 k

Wie kommt man auf den Schritt

40 - (x(8-x+5-x)) ?

Wie kommt man auf den Schritt
40 - (x(8-x+5-x)) ?

Sei \(F_R\) die Fläche des Rechtecks und \(F_1\) und \(F_2\) die Flächen der Dreiecke, so ist die Fläche \(F_P\) des Parallelogramms

$$\begin{aligned}F_P &= F_R - 2F_1  - 2F_2 \\ &= 5 \cdot 8 - 2\cdot\left( \frac 12 (8-x)x\right) - 2\cdot\left( \frac 12 (5-x) x\right) \\ &= 40 - (8-x)x - (5-x) x \\ &= 40 - ((8-x) + (5-x))x \\ &= 40 - (8-x + 5-x)x \\ &= 40 - (13-2x)x \\ &= 40 - 13x + 2x^2\end{aligned}$$

0 Daumen

Ich folge dem Hinweis. Die vier Dreiecke, von denen da die Rede ist haben zweimal die Fächeninhalte x(5-x)/2 und zweimal x(8-x)/2. Dann hat das Parallelogramm den Flächeninhalt f(x) = 40 - x(5-x) - x(8-x). Umformen und Nullstelle der Ableitung suchen. Für x = 3,25 hat das Parallelogramm den kleinsten Flächeninhalt.

von 123 k 🚀
0 Daumen

Die Frage ist zwar alt, aber weil mir dazu folgende Antwort eingefallen ist ...

Betrachte man zunächst mal folgendes CindyJS-Applet:

https://jsfiddle.net/WernerSalomon/578qzxrm/11/

Wenn man den (gelben) Punkt \(P\) mit der Maus verschiebt, verändert man den Wert von \(x=|PB|\) (rot) und damit die Fläche des Parallelogramms. Rechts daneben habe ich das Rechteck \(ABCD\) mit \(a=8\) und  \(b=5\) als \(A'B'C'D'\) noch einmal gezeichnet, aber die Flächen anders aufgeteilt. Denkt man sich die Dreiecke \(\triangle APS\) und \(\triangle QCR\) in das Rechteck \(S_1Q'C'R'\) sowie die Dreiecke \(\triangle PBQ\) und \(\triangle SRD\) in das Rechteck \(P'B'Q'S_2\) verlegt, so folgt daraus, dass die verbleibende grüne Fläche genauso groß sein muss, wie die des Parallelogramms \(PQRS\).

Mit Verschieben des Punktes \(P\) nach links, verkleinert sich das grüne Sechseck auf der Strecke \(|P'S_2|\) und \(|S_2S_1|\) (blau). Gleichzeitig vergrößert sich seine Fläche an der Seite \(|S_1R'|\) (gelb). Ein Extremum ist genau dann erreicht, wenn sich dieses Vergrößern und Verkleinern die Waage hält - also$$\begin{aligned}|P'S_2| + |S_2S_1| &= |S_1R'| \\ (b-x) + (a-2x)&= x \\  a+b &= 4x \\ \implies x &= \frac 14(a+b) = \frac 14(8+5) = \frac {13}4\\\end{aligned}$$kein Ableiten, keine quadratische Gleichung und man kann die Optimierungsaufgabe trotzdem lösen ;-)

Gruß Werner

von 48 k

Auf was für Ideen du immer wieder kommst, alle Achtung. 

@Silvia: Danke für das Feedback :-)

Ein anderes Problem?

Stell deine Frage

Willkommen bei der Mathelounge! Stell deine Frage einfach und kostenlos

x
Made by a lovely community